How can I prove that $(a_1+a_2+dotsb+a_n)(frac{1}{a_1}+frac{1}{a_2}+dotsb+frac{1}{a_n})geq n^2$ [duplicate]












3












$begingroup$



This question already has an answer here:




  • Proof that $left(sum^n_{k=1}x_kright)left(sum^n_{k=1}y_kright)geq n^2$

    7 answers




I've been struggling for several hours, trying to prove this horrible inequality:
$(a_1+a_2+dotsb+a_n)left(frac{1}{a_1}+frac{1}{a_2}+dotsb+frac{1}{a_n}right)geq n^2$.



Where each $a_i$'s are positive and $n$ is a natural number.



First I tried the usual "mathematical induction" method, but it made no avail, since I could not show it would be true if $n=k+1$.



Suppose the inequality holds true when $n=k$, i.e.,



$(a_1+a_2+dotsb+a_k)left(frac{1}{a_1}+frac{1}{a_2}+dotsb+frac{1}{a_k}right)geq n^2$.



This is true if and only if



$(a_1+a_2+dotsb+a_k+a_{k+1})left(frac{1}{a_1}+frac{1}{a_2}+dotsb+frac{1}{a_k}+frac{1}{a_{k+1}}right) -a_{k+1}left(frac{1}{a_1}+dotsb+frac{1}{a_k}right)-frac{1}{a_{k+1}}(a_1+dotsb+a_k)-frac{a_{k+1}}{a_{k+1}} geq n^2$.



And I got stuck here.



The question looks like I have to use AM-GM inequality at some point, but I do not have a clue. Any small hints and clues will be appreciated.










share|cite|improve this question











$endgroup$



marked as duplicate by Arnaud D., qwr, Martin R, Asaf Karagila 2 hours ago


This question has been asked before and already has an answer. If those answers do not fully address your question, please ask a new question.


















  • $begingroup$
    Conditions on $a_i$?
    $endgroup$
    – Parcly Taxel
    8 hours ago










  • $begingroup$
    whoa, I forgot the most important info there. They are all positive, and n is a natural number.
    $endgroup$
    – Ko Byeongmin
    8 hours ago






  • 4




    $begingroup$
    Try Cauchy-Schwarz inequality?
    $endgroup$
    – Sik Feng Cheong
    8 hours ago










  • $begingroup$
    Now I get it, I learn something new every day!! Thanks a lot :D
    $endgroup$
    – Ko Byeongmin
    8 hours ago
















3












$begingroup$



This question already has an answer here:




  • Proof that $left(sum^n_{k=1}x_kright)left(sum^n_{k=1}y_kright)geq n^2$

    7 answers




I've been struggling for several hours, trying to prove this horrible inequality:
$(a_1+a_2+dotsb+a_n)left(frac{1}{a_1}+frac{1}{a_2}+dotsb+frac{1}{a_n}right)geq n^2$.



Where each $a_i$'s are positive and $n$ is a natural number.



First I tried the usual "mathematical induction" method, but it made no avail, since I could not show it would be true if $n=k+1$.



Suppose the inequality holds true when $n=k$, i.e.,



$(a_1+a_2+dotsb+a_k)left(frac{1}{a_1}+frac{1}{a_2}+dotsb+frac{1}{a_k}right)geq n^2$.



This is true if and only if



$(a_1+a_2+dotsb+a_k+a_{k+1})left(frac{1}{a_1}+frac{1}{a_2}+dotsb+frac{1}{a_k}+frac{1}{a_{k+1}}right) -a_{k+1}left(frac{1}{a_1}+dotsb+frac{1}{a_k}right)-frac{1}{a_{k+1}}(a_1+dotsb+a_k)-frac{a_{k+1}}{a_{k+1}} geq n^2$.



And I got stuck here.



The question looks like I have to use AM-GM inequality at some point, but I do not have a clue. Any small hints and clues will be appreciated.










share|cite|improve this question











$endgroup$



marked as duplicate by Arnaud D., qwr, Martin R, Asaf Karagila 2 hours ago


This question has been asked before and already has an answer. If those answers do not fully address your question, please ask a new question.


















  • $begingroup$
    Conditions on $a_i$?
    $endgroup$
    – Parcly Taxel
    8 hours ago










  • $begingroup$
    whoa, I forgot the most important info there. They are all positive, and n is a natural number.
    $endgroup$
    – Ko Byeongmin
    8 hours ago






  • 4




    $begingroup$
    Try Cauchy-Schwarz inequality?
    $endgroup$
    – Sik Feng Cheong
    8 hours ago










  • $begingroup$
    Now I get it, I learn something new every day!! Thanks a lot :D
    $endgroup$
    – Ko Byeongmin
    8 hours ago














3












3








3


1



$begingroup$



This question already has an answer here:




  • Proof that $left(sum^n_{k=1}x_kright)left(sum^n_{k=1}y_kright)geq n^2$

    7 answers




I've been struggling for several hours, trying to prove this horrible inequality:
$(a_1+a_2+dotsb+a_n)left(frac{1}{a_1}+frac{1}{a_2}+dotsb+frac{1}{a_n}right)geq n^2$.



Where each $a_i$'s are positive and $n$ is a natural number.



First I tried the usual "mathematical induction" method, but it made no avail, since I could not show it would be true if $n=k+1$.



Suppose the inequality holds true when $n=k$, i.e.,



$(a_1+a_2+dotsb+a_k)left(frac{1}{a_1}+frac{1}{a_2}+dotsb+frac{1}{a_k}right)geq n^2$.



This is true if and only if



$(a_1+a_2+dotsb+a_k+a_{k+1})left(frac{1}{a_1}+frac{1}{a_2}+dotsb+frac{1}{a_k}+frac{1}{a_{k+1}}right) -a_{k+1}left(frac{1}{a_1}+dotsb+frac{1}{a_k}right)-frac{1}{a_{k+1}}(a_1+dotsb+a_k)-frac{a_{k+1}}{a_{k+1}} geq n^2$.



And I got stuck here.



The question looks like I have to use AM-GM inequality at some point, but I do not have a clue. Any small hints and clues will be appreciated.










share|cite|improve this question











$endgroup$





This question already has an answer here:




  • Proof that $left(sum^n_{k=1}x_kright)left(sum^n_{k=1}y_kright)geq n^2$

    7 answers




I've been struggling for several hours, trying to prove this horrible inequality:
$(a_1+a_2+dotsb+a_n)left(frac{1}{a_1}+frac{1}{a_2}+dotsb+frac{1}{a_n}right)geq n^2$.



Where each $a_i$'s are positive and $n$ is a natural number.



First I tried the usual "mathematical induction" method, but it made no avail, since I could not show it would be true if $n=k+1$.



Suppose the inequality holds true when $n=k$, i.e.,



$(a_1+a_2+dotsb+a_k)left(frac{1}{a_1}+frac{1}{a_2}+dotsb+frac{1}{a_k}right)geq n^2$.



This is true if and only if



$(a_1+a_2+dotsb+a_k+a_{k+1})left(frac{1}{a_1}+frac{1}{a_2}+dotsb+frac{1}{a_k}+frac{1}{a_{k+1}}right) -a_{k+1}left(frac{1}{a_1}+dotsb+frac{1}{a_k}right)-frac{1}{a_{k+1}}(a_1+dotsb+a_k)-frac{a_{k+1}}{a_{k+1}} geq n^2$.



And I got stuck here.



The question looks like I have to use AM-GM inequality at some point, but I do not have a clue. Any small hints and clues will be appreciated.





This question already has an answer here:




  • Proof that $left(sum^n_{k=1}x_kright)left(sum^n_{k=1}y_kright)geq n^2$

    7 answers








analysis inequality






share|cite|improve this question















share|cite|improve this question













share|cite|improve this question




share|cite|improve this question








edited 3 hours ago









Asaf Karagila

306k33438769




306k33438769










asked 8 hours ago









Ko ByeongminKo Byeongmin

1546




1546




marked as duplicate by Arnaud D., qwr, Martin R, Asaf Karagila 2 hours ago


This question has been asked before and already has an answer. If those answers do not fully address your question, please ask a new question.









marked as duplicate by Arnaud D., qwr, Martin R, Asaf Karagila 2 hours ago


This question has been asked before and already has an answer. If those answers do not fully address your question, please ask a new question.














  • $begingroup$
    Conditions on $a_i$?
    $endgroup$
    – Parcly Taxel
    8 hours ago










  • $begingroup$
    whoa, I forgot the most important info there. They are all positive, and n is a natural number.
    $endgroup$
    – Ko Byeongmin
    8 hours ago






  • 4




    $begingroup$
    Try Cauchy-Schwarz inequality?
    $endgroup$
    – Sik Feng Cheong
    8 hours ago










  • $begingroup$
    Now I get it, I learn something new every day!! Thanks a lot :D
    $endgroup$
    – Ko Byeongmin
    8 hours ago


















  • $begingroup$
    Conditions on $a_i$?
    $endgroup$
    – Parcly Taxel
    8 hours ago










  • $begingroup$
    whoa, I forgot the most important info there. They are all positive, and n is a natural number.
    $endgroup$
    – Ko Byeongmin
    8 hours ago






  • 4




    $begingroup$
    Try Cauchy-Schwarz inequality?
    $endgroup$
    – Sik Feng Cheong
    8 hours ago










  • $begingroup$
    Now I get it, I learn something new every day!! Thanks a lot :D
    $endgroup$
    – Ko Byeongmin
    8 hours ago
















$begingroup$
Conditions on $a_i$?
$endgroup$
– Parcly Taxel
8 hours ago




$begingroup$
Conditions on $a_i$?
$endgroup$
– Parcly Taxel
8 hours ago












$begingroup$
whoa, I forgot the most important info there. They are all positive, and n is a natural number.
$endgroup$
– Ko Byeongmin
8 hours ago




$begingroup$
whoa, I forgot the most important info there. They are all positive, and n is a natural number.
$endgroup$
– Ko Byeongmin
8 hours ago




4




4




$begingroup$
Try Cauchy-Schwarz inequality?
$endgroup$
– Sik Feng Cheong
8 hours ago




$begingroup$
Try Cauchy-Schwarz inequality?
$endgroup$
– Sik Feng Cheong
8 hours ago












$begingroup$
Now I get it, I learn something new every day!! Thanks a lot :D
$endgroup$
– Ko Byeongmin
8 hours ago




$begingroup$
Now I get it, I learn something new every day!! Thanks a lot :D
$endgroup$
– Ko Byeongmin
8 hours ago










3 Answers
3






active

oldest

votes


















12












$begingroup$

Hint: AM-GM implies
$$
a_1+a_2+cdots +a_nge nsqrt[n]{a_1a_2cdots a_n}
$$
and $$
frac1{a_1}+frac1{a_2}+cdots +frac1{a_n}ge frac{n}{sqrt[n]{a_1a_2cdots a_n}}.
$$






share|cite|improve this answer









$endgroup$













  • $begingroup$
    That's a really strong hint, it looks like an answer
    $endgroup$
    – enedil
    6 hours ago










  • $begingroup$
    You may be right.. It makes the remaining step look so trivial, so it can be justly regarded as an answer.
    $endgroup$
    – Song
    6 hours ago



















9












$begingroup$

It is AM-HM inequality
$$frac{a_1+a_2+a_3+...+a_n}{n}geq frac{n}{frac{1}{a_1}+frac{1}{a_2}+frac{1}{a_3}+...+frac{1}{a_n}}$$






share|cite|improve this answer









$endgroup$





















    1












    $begingroup$

    Here is the proof by induction
    that you wanted.



    I added a more exact
    version of
    the identity used
    in the proof
    at the end.



    Let
    $s_n
    =u_nv_n
    $

    where
    $u_n=sum_{k=1}^n a_k,
    v_n= sum_{k=1}^n dfrac1{a_k}
    $
    .
    Then,
    assuming
    $s_n ge n^2$,



    $begin{array}\
    s_{n+1}
    &=u_{n+1}v_{n+1}\
    &=(u_n+a_{n+1}) (v_n+dfrac1{a_{n+1}})\
    &=u_nv_n+u_ndfrac1{a_{n+1}}+a_{n+1}v_n+1\
    &=s_n+u_ndfrac1{a_{n+1}}+a_{n+1}v_n+1\
    &ge n^2+u_ndfrac1{a_{n+1}}+a_{n+1}v_n+1\
    end{array}
    $



    So it is sufficient
    to show that
    $u_ndfrac1{a_{n+1}}+v_na_{n+1}
    ge 2n
    $
    .



    By simple algebra,
    if $a, b ge 0$ then
    $a+b
    ge 2sqrt{ab}
    $
    .
    (Rewrite as
    $(sqrt{a}-sqrt{b})^2ge 0$
    or,
    as an identity,
    $a+b
    =2ab+(sqrt{a}-sqrt{b})^2$
    .)



    Therefore



    $begin{array}\
    u_ndfrac1{a_{n+1}}+v_na_{n+1}
    &ge sqrt{(u_ndfrac1{a_{n+1}})(v_na_{n+1})}\
    &= sqrt{u_nv_n}\
    &=2sqrt{s_n}\
    &ge 2sqrt{n^2}
    qquadtext{by the induction hypothesis}\
    &=2n\
    end{array}
    $



    and we are done.



    I find it interesting that
    $s_n ge n^2$
    is used twice in the
    induction step.



    Note that,
    if we use the identity above,
    $a+b
    =2ab+(sqrt{a}-sqrt{b})^2$
    ,
    we get this:



    $begin{array}\
    s_{n+1}
    &=s_n+u_ndfrac1{a_{n+1}}+a_{n+1}v_n+1\
    &=s_n+2sqrt{u_ndfrac1{a_{n+1}}a_{n+1}v_n}+1+(sqrt{u_ndfrac1{a_{n+1}}}-sqrt{a_{n+1}v_n})^2\
    &=s_n+2sqrt{s_n}+1+dfrac1{sqrt{a_{n+1}}}(sqrt{u_n}-a_{n+1}sqrt{v_n})^2\
    &=(sqrt{s_n}+1)^2+dfrac1{sqrt{a_{n+1}}}(sqrt{u_n}-a_{n+1}sqrt{v_n})^2\
    &ge(sqrt{s_n}+1)^2\
    end{array}
    $



    with equality
    if and only if
    $a_{n+1}
    =sqrt{dfrac{u_n}{v_n}}
    =sqrt{dfrac{sum_{k=1}^n a_k}{sum_{k=1}^n dfrac1{a_k}}}
    $
    .






    share|cite|improve this answer











    $endgroup$




















      3 Answers
      3






      active

      oldest

      votes








      3 Answers
      3






      active

      oldest

      votes









      active

      oldest

      votes






      active

      oldest

      votes









      12












      $begingroup$

      Hint: AM-GM implies
      $$
      a_1+a_2+cdots +a_nge nsqrt[n]{a_1a_2cdots a_n}
      $$
      and $$
      frac1{a_1}+frac1{a_2}+cdots +frac1{a_n}ge frac{n}{sqrt[n]{a_1a_2cdots a_n}}.
      $$






      share|cite|improve this answer









      $endgroup$













      • $begingroup$
        That's a really strong hint, it looks like an answer
        $endgroup$
        – enedil
        6 hours ago










      • $begingroup$
        You may be right.. It makes the remaining step look so trivial, so it can be justly regarded as an answer.
        $endgroup$
        – Song
        6 hours ago
















      12












      $begingroup$

      Hint: AM-GM implies
      $$
      a_1+a_2+cdots +a_nge nsqrt[n]{a_1a_2cdots a_n}
      $$
      and $$
      frac1{a_1}+frac1{a_2}+cdots +frac1{a_n}ge frac{n}{sqrt[n]{a_1a_2cdots a_n}}.
      $$






      share|cite|improve this answer









      $endgroup$













      • $begingroup$
        That's a really strong hint, it looks like an answer
        $endgroup$
        – enedil
        6 hours ago










      • $begingroup$
        You may be right.. It makes the remaining step look so trivial, so it can be justly regarded as an answer.
        $endgroup$
        – Song
        6 hours ago














      12












      12








      12





      $begingroup$

      Hint: AM-GM implies
      $$
      a_1+a_2+cdots +a_nge nsqrt[n]{a_1a_2cdots a_n}
      $$
      and $$
      frac1{a_1}+frac1{a_2}+cdots +frac1{a_n}ge frac{n}{sqrt[n]{a_1a_2cdots a_n}}.
      $$






      share|cite|improve this answer









      $endgroup$



      Hint: AM-GM implies
      $$
      a_1+a_2+cdots +a_nge nsqrt[n]{a_1a_2cdots a_n}
      $$
      and $$
      frac1{a_1}+frac1{a_2}+cdots +frac1{a_n}ge frac{n}{sqrt[n]{a_1a_2cdots a_n}}.
      $$







      share|cite|improve this answer












      share|cite|improve this answer



      share|cite|improve this answer










      answered 8 hours ago









      SongSong

      17.2k21246




      17.2k21246












      • $begingroup$
        That's a really strong hint, it looks like an answer
        $endgroup$
        – enedil
        6 hours ago










      • $begingroup$
        You may be right.. It makes the remaining step look so trivial, so it can be justly regarded as an answer.
        $endgroup$
        – Song
        6 hours ago


















      • $begingroup$
        That's a really strong hint, it looks like an answer
        $endgroup$
        – enedil
        6 hours ago










      • $begingroup$
        You may be right.. It makes the remaining step look so trivial, so it can be justly regarded as an answer.
        $endgroup$
        – Song
        6 hours ago
















      $begingroup$
      That's a really strong hint, it looks like an answer
      $endgroup$
      – enedil
      6 hours ago




      $begingroup$
      That's a really strong hint, it looks like an answer
      $endgroup$
      – enedil
      6 hours ago












      $begingroup$
      You may be right.. It makes the remaining step look so trivial, so it can be justly regarded as an answer.
      $endgroup$
      – Song
      6 hours ago




      $begingroup$
      You may be right.. It makes the remaining step look so trivial, so it can be justly regarded as an answer.
      $endgroup$
      – Song
      6 hours ago











      9












      $begingroup$

      It is AM-HM inequality
      $$frac{a_1+a_2+a_3+...+a_n}{n}geq frac{n}{frac{1}{a_1}+frac{1}{a_2}+frac{1}{a_3}+...+frac{1}{a_n}}$$






      share|cite|improve this answer









      $endgroup$


















        9












        $begingroup$

        It is AM-HM inequality
        $$frac{a_1+a_2+a_3+...+a_n}{n}geq frac{n}{frac{1}{a_1}+frac{1}{a_2}+frac{1}{a_3}+...+frac{1}{a_n}}$$






        share|cite|improve this answer









        $endgroup$
















          9












          9








          9





          $begingroup$

          It is AM-HM inequality
          $$frac{a_1+a_2+a_3+...+a_n}{n}geq frac{n}{frac{1}{a_1}+frac{1}{a_2}+frac{1}{a_3}+...+frac{1}{a_n}}$$






          share|cite|improve this answer









          $endgroup$



          It is AM-HM inequality
          $$frac{a_1+a_2+a_3+...+a_n}{n}geq frac{n}{frac{1}{a_1}+frac{1}{a_2}+frac{1}{a_3}+...+frac{1}{a_n}}$$







          share|cite|improve this answer












          share|cite|improve this answer



          share|cite|improve this answer










          answered 8 hours ago









          Dr. Sonnhard GraubnerDr. Sonnhard Graubner

          77.7k42866




          77.7k42866























              1












              $begingroup$

              Here is the proof by induction
              that you wanted.



              I added a more exact
              version of
              the identity used
              in the proof
              at the end.



              Let
              $s_n
              =u_nv_n
              $

              where
              $u_n=sum_{k=1}^n a_k,
              v_n= sum_{k=1}^n dfrac1{a_k}
              $
              .
              Then,
              assuming
              $s_n ge n^2$,



              $begin{array}\
              s_{n+1}
              &=u_{n+1}v_{n+1}\
              &=(u_n+a_{n+1}) (v_n+dfrac1{a_{n+1}})\
              &=u_nv_n+u_ndfrac1{a_{n+1}}+a_{n+1}v_n+1\
              &=s_n+u_ndfrac1{a_{n+1}}+a_{n+1}v_n+1\
              &ge n^2+u_ndfrac1{a_{n+1}}+a_{n+1}v_n+1\
              end{array}
              $



              So it is sufficient
              to show that
              $u_ndfrac1{a_{n+1}}+v_na_{n+1}
              ge 2n
              $
              .



              By simple algebra,
              if $a, b ge 0$ then
              $a+b
              ge 2sqrt{ab}
              $
              .
              (Rewrite as
              $(sqrt{a}-sqrt{b})^2ge 0$
              or,
              as an identity,
              $a+b
              =2ab+(sqrt{a}-sqrt{b})^2$
              .)



              Therefore



              $begin{array}\
              u_ndfrac1{a_{n+1}}+v_na_{n+1}
              &ge sqrt{(u_ndfrac1{a_{n+1}})(v_na_{n+1})}\
              &= sqrt{u_nv_n}\
              &=2sqrt{s_n}\
              &ge 2sqrt{n^2}
              qquadtext{by the induction hypothesis}\
              &=2n\
              end{array}
              $



              and we are done.



              I find it interesting that
              $s_n ge n^2$
              is used twice in the
              induction step.



              Note that,
              if we use the identity above,
              $a+b
              =2ab+(sqrt{a}-sqrt{b})^2$
              ,
              we get this:



              $begin{array}\
              s_{n+1}
              &=s_n+u_ndfrac1{a_{n+1}}+a_{n+1}v_n+1\
              &=s_n+2sqrt{u_ndfrac1{a_{n+1}}a_{n+1}v_n}+1+(sqrt{u_ndfrac1{a_{n+1}}}-sqrt{a_{n+1}v_n})^2\
              &=s_n+2sqrt{s_n}+1+dfrac1{sqrt{a_{n+1}}}(sqrt{u_n}-a_{n+1}sqrt{v_n})^2\
              &=(sqrt{s_n}+1)^2+dfrac1{sqrt{a_{n+1}}}(sqrt{u_n}-a_{n+1}sqrt{v_n})^2\
              &ge(sqrt{s_n}+1)^2\
              end{array}
              $



              with equality
              if and only if
              $a_{n+1}
              =sqrt{dfrac{u_n}{v_n}}
              =sqrt{dfrac{sum_{k=1}^n a_k}{sum_{k=1}^n dfrac1{a_k}}}
              $
              .






              share|cite|improve this answer











              $endgroup$


















                1












                $begingroup$

                Here is the proof by induction
                that you wanted.



                I added a more exact
                version of
                the identity used
                in the proof
                at the end.



                Let
                $s_n
                =u_nv_n
                $

                where
                $u_n=sum_{k=1}^n a_k,
                v_n= sum_{k=1}^n dfrac1{a_k}
                $
                .
                Then,
                assuming
                $s_n ge n^2$,



                $begin{array}\
                s_{n+1}
                &=u_{n+1}v_{n+1}\
                &=(u_n+a_{n+1}) (v_n+dfrac1{a_{n+1}})\
                &=u_nv_n+u_ndfrac1{a_{n+1}}+a_{n+1}v_n+1\
                &=s_n+u_ndfrac1{a_{n+1}}+a_{n+1}v_n+1\
                &ge n^2+u_ndfrac1{a_{n+1}}+a_{n+1}v_n+1\
                end{array}
                $



                So it is sufficient
                to show that
                $u_ndfrac1{a_{n+1}}+v_na_{n+1}
                ge 2n
                $
                .



                By simple algebra,
                if $a, b ge 0$ then
                $a+b
                ge 2sqrt{ab}
                $
                .
                (Rewrite as
                $(sqrt{a}-sqrt{b})^2ge 0$
                or,
                as an identity,
                $a+b
                =2ab+(sqrt{a}-sqrt{b})^2$
                .)



                Therefore



                $begin{array}\
                u_ndfrac1{a_{n+1}}+v_na_{n+1}
                &ge sqrt{(u_ndfrac1{a_{n+1}})(v_na_{n+1})}\
                &= sqrt{u_nv_n}\
                &=2sqrt{s_n}\
                &ge 2sqrt{n^2}
                qquadtext{by the induction hypothesis}\
                &=2n\
                end{array}
                $



                and we are done.



                I find it interesting that
                $s_n ge n^2$
                is used twice in the
                induction step.



                Note that,
                if we use the identity above,
                $a+b
                =2ab+(sqrt{a}-sqrt{b})^2$
                ,
                we get this:



                $begin{array}\
                s_{n+1}
                &=s_n+u_ndfrac1{a_{n+1}}+a_{n+1}v_n+1\
                &=s_n+2sqrt{u_ndfrac1{a_{n+1}}a_{n+1}v_n}+1+(sqrt{u_ndfrac1{a_{n+1}}}-sqrt{a_{n+1}v_n})^2\
                &=s_n+2sqrt{s_n}+1+dfrac1{sqrt{a_{n+1}}}(sqrt{u_n}-a_{n+1}sqrt{v_n})^2\
                &=(sqrt{s_n}+1)^2+dfrac1{sqrt{a_{n+1}}}(sqrt{u_n}-a_{n+1}sqrt{v_n})^2\
                &ge(sqrt{s_n}+1)^2\
                end{array}
                $



                with equality
                if and only if
                $a_{n+1}
                =sqrt{dfrac{u_n}{v_n}}
                =sqrt{dfrac{sum_{k=1}^n a_k}{sum_{k=1}^n dfrac1{a_k}}}
                $
                .






                share|cite|improve this answer











                $endgroup$
















                  1












                  1








                  1





                  $begingroup$

                  Here is the proof by induction
                  that you wanted.



                  I added a more exact
                  version of
                  the identity used
                  in the proof
                  at the end.



                  Let
                  $s_n
                  =u_nv_n
                  $

                  where
                  $u_n=sum_{k=1}^n a_k,
                  v_n= sum_{k=1}^n dfrac1{a_k}
                  $
                  .
                  Then,
                  assuming
                  $s_n ge n^2$,



                  $begin{array}\
                  s_{n+1}
                  &=u_{n+1}v_{n+1}\
                  &=(u_n+a_{n+1}) (v_n+dfrac1{a_{n+1}})\
                  &=u_nv_n+u_ndfrac1{a_{n+1}}+a_{n+1}v_n+1\
                  &=s_n+u_ndfrac1{a_{n+1}}+a_{n+1}v_n+1\
                  &ge n^2+u_ndfrac1{a_{n+1}}+a_{n+1}v_n+1\
                  end{array}
                  $



                  So it is sufficient
                  to show that
                  $u_ndfrac1{a_{n+1}}+v_na_{n+1}
                  ge 2n
                  $
                  .



                  By simple algebra,
                  if $a, b ge 0$ then
                  $a+b
                  ge 2sqrt{ab}
                  $
                  .
                  (Rewrite as
                  $(sqrt{a}-sqrt{b})^2ge 0$
                  or,
                  as an identity,
                  $a+b
                  =2ab+(sqrt{a}-sqrt{b})^2$
                  .)



                  Therefore



                  $begin{array}\
                  u_ndfrac1{a_{n+1}}+v_na_{n+1}
                  &ge sqrt{(u_ndfrac1{a_{n+1}})(v_na_{n+1})}\
                  &= sqrt{u_nv_n}\
                  &=2sqrt{s_n}\
                  &ge 2sqrt{n^2}
                  qquadtext{by the induction hypothesis}\
                  &=2n\
                  end{array}
                  $



                  and we are done.



                  I find it interesting that
                  $s_n ge n^2$
                  is used twice in the
                  induction step.



                  Note that,
                  if we use the identity above,
                  $a+b
                  =2ab+(sqrt{a}-sqrt{b})^2$
                  ,
                  we get this:



                  $begin{array}\
                  s_{n+1}
                  &=s_n+u_ndfrac1{a_{n+1}}+a_{n+1}v_n+1\
                  &=s_n+2sqrt{u_ndfrac1{a_{n+1}}a_{n+1}v_n}+1+(sqrt{u_ndfrac1{a_{n+1}}}-sqrt{a_{n+1}v_n})^2\
                  &=s_n+2sqrt{s_n}+1+dfrac1{sqrt{a_{n+1}}}(sqrt{u_n}-a_{n+1}sqrt{v_n})^2\
                  &=(sqrt{s_n}+1)^2+dfrac1{sqrt{a_{n+1}}}(sqrt{u_n}-a_{n+1}sqrt{v_n})^2\
                  &ge(sqrt{s_n}+1)^2\
                  end{array}
                  $



                  with equality
                  if and only if
                  $a_{n+1}
                  =sqrt{dfrac{u_n}{v_n}}
                  =sqrt{dfrac{sum_{k=1}^n a_k}{sum_{k=1}^n dfrac1{a_k}}}
                  $
                  .






                  share|cite|improve this answer











                  $endgroup$



                  Here is the proof by induction
                  that you wanted.



                  I added a more exact
                  version of
                  the identity used
                  in the proof
                  at the end.



                  Let
                  $s_n
                  =u_nv_n
                  $

                  where
                  $u_n=sum_{k=1}^n a_k,
                  v_n= sum_{k=1}^n dfrac1{a_k}
                  $
                  .
                  Then,
                  assuming
                  $s_n ge n^2$,



                  $begin{array}\
                  s_{n+1}
                  &=u_{n+1}v_{n+1}\
                  &=(u_n+a_{n+1}) (v_n+dfrac1{a_{n+1}})\
                  &=u_nv_n+u_ndfrac1{a_{n+1}}+a_{n+1}v_n+1\
                  &=s_n+u_ndfrac1{a_{n+1}}+a_{n+1}v_n+1\
                  &ge n^2+u_ndfrac1{a_{n+1}}+a_{n+1}v_n+1\
                  end{array}
                  $



                  So it is sufficient
                  to show that
                  $u_ndfrac1{a_{n+1}}+v_na_{n+1}
                  ge 2n
                  $
                  .



                  By simple algebra,
                  if $a, b ge 0$ then
                  $a+b
                  ge 2sqrt{ab}
                  $
                  .
                  (Rewrite as
                  $(sqrt{a}-sqrt{b})^2ge 0$
                  or,
                  as an identity,
                  $a+b
                  =2ab+(sqrt{a}-sqrt{b})^2$
                  .)



                  Therefore



                  $begin{array}\
                  u_ndfrac1{a_{n+1}}+v_na_{n+1}
                  &ge sqrt{(u_ndfrac1{a_{n+1}})(v_na_{n+1})}\
                  &= sqrt{u_nv_n}\
                  &=2sqrt{s_n}\
                  &ge 2sqrt{n^2}
                  qquadtext{by the induction hypothesis}\
                  &=2n\
                  end{array}
                  $



                  and we are done.



                  I find it interesting that
                  $s_n ge n^2$
                  is used twice in the
                  induction step.



                  Note that,
                  if we use the identity above,
                  $a+b
                  =2ab+(sqrt{a}-sqrt{b})^2$
                  ,
                  we get this:



                  $begin{array}\
                  s_{n+1}
                  &=s_n+u_ndfrac1{a_{n+1}}+a_{n+1}v_n+1\
                  &=s_n+2sqrt{u_ndfrac1{a_{n+1}}a_{n+1}v_n}+1+(sqrt{u_ndfrac1{a_{n+1}}}-sqrt{a_{n+1}v_n})^2\
                  &=s_n+2sqrt{s_n}+1+dfrac1{sqrt{a_{n+1}}}(sqrt{u_n}-a_{n+1}sqrt{v_n})^2\
                  &=(sqrt{s_n}+1)^2+dfrac1{sqrt{a_{n+1}}}(sqrt{u_n}-a_{n+1}sqrt{v_n})^2\
                  &ge(sqrt{s_n}+1)^2\
                  end{array}
                  $



                  with equality
                  if and only if
                  $a_{n+1}
                  =sqrt{dfrac{u_n}{v_n}}
                  =sqrt{dfrac{sum_{k=1}^n a_k}{sum_{k=1}^n dfrac1{a_k}}}
                  $
                  .







                  share|cite|improve this answer














                  share|cite|improve this answer



                  share|cite|improve this answer








                  edited 37 mins ago

























                  answered 3 hours ago









                  marty cohenmarty cohen

                  74.2k549128




                  74.2k549128















                      Popular posts from this blog

                      How did Captain America manage to do this?

                      迪纳利

                      南乌拉尔铁路局